LSAT and Law School Admissions Forum

Get expert LSAT preparation and law school admissions advice from PowerScore Test Preparation.

 Administrator
PowerScore Staff
  • PowerScore Staff
  • Posts: 8916
  • Joined: Feb 02, 2011
|
#37028
Please post below with any questions!
 ScholesFan
  • Posts: 13
  • Joined: Dec 29, 2018
|
#61517
Hi PowerScore,

I diagrammed the stimulus from this question, but it looks a little funky. I want to make sure I'm doing it correctly. Here's what I've got:

If I Borrow something & Promise to return it & it's Not Difficult to return & I don't have permission to return it late —> I should Return it.

Does it look like I'm understanding the conditional reasoning correctly? It's just such a long trail of sufficient conditions that I'm not sure I needed to include all of them.

Thanks!
 ScholesFan
  • Posts: 13
  • Joined: Dec 29, 2018
|
#61526
Awesome — thank you!
 Boudreaux
  • Posts: 9
  • Joined: Jan 04, 2019
|
#62042
Just for clarification... I chose E, but as I'm checking my answers, I'm wondering why I ruled out C. Could you provide some reasoning for why C is incorrect? Thanks
 James Finch
PowerScore Staff
  • PowerScore Staff
  • Posts: 943
  • Joined: Sep 06, 2017
|
#62065
Hi Boudreaux,

The principle we're given in the stimulus is that, in situations where a borrower has promised to return an item by a certain date, if it wouldn't be difficult to return it on time and the lender hasn't given permission to return it late, the item should be returned at the promised time. This can be diagrammed as:

Difficult (D) + Permission for Late Return (PLR) :arrow: Should Return on Time (SRT)

with a contrapositive of:

SRT :arrow: D or PLR

So to conclude that somebody should return an item on time, we need both conditions fulfilled. (C) fulfills the "not difficult" condition, but fails by having the borrower grant permission to return the item late.

Hope this clears things up!
 Boudreaux
  • Posts: 9
  • Joined: Jan 04, 2019
|
#62080
Got it. Thanks James.
 Naminyar
  • Posts: 15
  • Joined: Jun 28, 2018
|
#68139
Hello PowerScore,

Based on PowerScore question type classification, what type of question is this?
The question stem is “strengthen” however we are tasked to help the reasoning in an answer choice. Is this question still a strengthen question?
User avatar
 Dave Killoran
PowerScore Staff
  • PowerScore Staff
  • Posts: 5853
  • Joined: Mar 25, 2011
|
#68144
Hi Naminyar,

It's not a Strengthen stem, but this is a good example of how LSAC plays with your preconceived notions. So let's break it down:

Here's the full stem: "The principle stated above most helps to justify the reasoning in which one of the following arguments?"

From the start, we can tell it's a principle question: ""The principle stated above..." So, whatever the main stem turns out to be, this will have a PR overlay on it. Easy so far :-D

Next, they throw in something that should be familiar to everyone: "...most helps to justify the reasoning..." That would normally be a Strengthen stem (due to the "most"), and so many readers simply say, "Oh, it's a Strengthen-PR question. Got it!"

BUT, look how they framed that idea (italics added): "The principle stated above most helps to justify the reasoning in which one of the following arguments?" This means that an idea in the stimulus is being used to determine the answer, which is a classic First Family information arrangement (arrow pointing down from the stimulus to the answers). Strengthen questions are Second Family (arrow point up), so we know right now this isn't a Strengthen question.

In the First Family, what would be similar to "most helps to justify" an answer choice? Since its telling you to use the principle in the argument to "prove" an answer, that would be closest to Most Strongly Supported, and that's what this is ultimately. We'd put it down under Must-PR since Must includes MSS for us.

Overall, an excellent exercise in showing that you simply can't assume that the appearance of certain code words ("most helps to justify the reasoning" in this case, or "strengthen" in another popular example) automatically means you can classify the stem. Instead, you have to see what they do with that language.

Please let me know if that helps, thanks!
 Naminyar
  • Posts: 15
  • Joined: Jun 28, 2018
|
#68148
Crystal Clear!

Thanks Dave

Get the most out of your LSAT Prep Plus subscription.

Analyze and track your performance with our Testing and Analytics Package.